Вывод плотности Лагранжа для электромагнитного поля

При рассмотрении (специального) релятивистского электромагнитного поля я понимаю, что если принять лагранжеву плотность формы

л "=" 1 4 Ф мю ν Ф мю ν + 1 с Дж мю А мю

и, следуя уравнениям Эйлера-Лагранжа, восстанавливает уравнения Максвелла.

Существует ли вывод этого лагранжиана из первых принципов? Ссылка или объяснение будет принята с благодарностью!

С каких принципов вы хотите начать?

Ответы (5)

Абстрактный

Далее мы докажем, что согласованная плотность лагранжиана для электромагнитного поля в пустом пространстве равна

(045) л е м "=" ϵ 0 | | Е | | 2 с 2 | | Б | | 2 2 р ф + Дж А
то есть уравнения Эйлера-Лангранжа, полученные из этого лагранжиана, являются уравнениями Максвелла для электромагнитного поля.

Эта лагранжева плотность получается методом проб и ошибок (1) , а не путем угадывания.

1. Введение

Дифференциальные уравнения Максвелла электромагнитного поля в пустом пространстве имеют вид

(001а) × Е "=" Б т (001б) × Б "=" мю 0 Дж + 1 с 2 Е т (001с) Е "=" р ϵ 0 (001д) Б "=" 0
где Е "=" вектор напряженности электрического поля, Б "=" вектор плотности магнитного потока, р "=" плотность электрического заряда, Дж "=" вектор плотности электрического тока. Все величины являются функциями трех пространственных координат ( Икс 1 , Икс 2 , Икс 3 ) ( Икс , у , г ) и время т Икс 4 .

Из уравнения (001d) вектор магнитного потока Б может быть выражен как ротор векторного потенциала А

(002) Б "=" × А
и из (002) уравнение (001a) дает
(003) × ( Е + А т ) "=" 0
поэтому член в скобках может быть выражен как градиент скалярной функции
Е + А т "=" ф
то есть
(004) Е "=" ф А т
Итак, шесть скалярных переменных, компоненты векторов Е и Б , может быть выражен как функция 4 скалярных переменных, скалярный потенциал ф и три компоненты векторного потенциала А .

Вставка выражений Е и Б , уравнения (002) и (004) соответственно, в уравнениях (001b) и (001c) имеем

(005) × ( × А ) "=" мю 0 Дж + 1 с 2 т ( ф А т )

и
(006) 2 ф т ( А ) "=" р ϵ 0
При условии
(007) × ( × А ) "=" ( А ) 2 А
уравнение (005) дает
(008) 1 с 2 2 А т 2 2 А + ( А + 1 с 2 ф т ) "=" мю 0 Дж

2. Уравнения Эйлера-Лагранжа ЭМ поля.

Теперь наша основная задача — найти плотность лагранжиана л , функция четырех «координат поля» и их производных первого порядка

(009) л "=" л ( η ȷ , η ȷ , η ȷ ) ( ȷ "=" 1 , 2 , 3 , 4 )
так что четыре уравнения скалярного электромагнитного поля (006) и (008) выводятся из уравнений Лагранжа
(010) т [ л ( η ȷ т ) ] + к "=" 1 к "=" 3 Икс к [ л ( η ȷ Икс к ) ] л η ȷ "=" 0 , ( ȷ "=" 1 , 2 , 3 , 4 )
упрощенный в обозначении до
(011) т ( л η ȷ ) + [ л ( η ȷ ) ] л η ȷ "=" 0 , ( ȷ "=" 1 , 2 , 3 , 4 )

Здесь лагранжева плотность л является функцией

  1. четыре «координаты поля»

(012.1) η 1 "=" А 1 ( Икс 1 , Икс 2 , Икс 3 , т ) (012.2) η 2 "=" А 2 ( Икс 1 , Икс 2 , Икс 3 , т ) (012.3) η 3 "=" А 3 ( Икс 1 , Икс 2 , Икс 3 , т ) (012.4) η 4 "=" ф ( Икс 1 , Икс 2 , Икс 3 , т )

  1. их производные по времени

(013.1) η 1 η 1 т "=" А 1 т А 1 (013.2) η 2 η 2 т "=" А 2 т А 2 (013.3) η 3 η 3 т "=" А 3 т А 3 (013.4) η 4 η 4 т "=" ф т ф

и

  1. их градиенты

(014) η 1 "=" А 1 , η 2 "=" А 2 , η 3 "=" А 3 , η 4 "=" ф

Выразим уравнения (006) и (008) в формах, аналогичных уравнениям Лагранжа (011)

(015) т ( А ) + ( ф ) ( р ϵ 0 ) "=" 0
и
(016) т ( А к т + ф Икс к ) + [ с 2 ( А Икс к А к ) ] Дж к ϵ 0 "=" 0
Уравнение Лагранжа (011) для ȷ "=" 4 , то есть для η 4 "=" ф , является
(017) т ( л ф ) + [ л ( ф ) ] л ф "=" 0

Сравнивая уравнения (015) и (017), заметим, что первое можно было бы вывести из второго, если

(018) л ф "=" А , л ( ф ) "=" ф , л ф "=" р ϵ 0

так что лагранжева плотность л должны содержать соответственно термины
(019) л α 1 ( А ) ф , л α 2 1 2 ф 2 , л α 3 р ф ϵ 0
и, следовательно, их сумма
(020) л α "=" л α 1 + л α 2 + л α 3 "=" ( А ) ф + 1 2 ф 2 р ф ϵ 0

Мы предполагаем, что подходящая лагранжева плотность л будет иметь форму

(021) л "=" л α + л β
и с тех пор л α дает уравнение (015), мы ожидаем, что л β , который необходимо определить, даст уравнения (016). Это ожидание было бы правильным, если бы уравнения (015) и (016) были разделены, например, если бы первое содержало ф -термины только и второй А -только термины. Но здесь это не так: л α как содержащий А -термы будут участвовать в построении уравнений (016) и, кроме того, л β будет участвовать в создании уравнения (015), возможно, взаимно разрушая производство уравнений, как мы ожидали. Но здесь мы следуем процедуре проб и ошибок, которая приведет к правильному ответу, как мы увидим далее.

Теперь уравнения Лагранжа (011) для ȷ "=" к "=" 1 , 2 , 3 , то есть для η к "=" А к , являются

(022) т ( л А к ) + [ л ( А к ) ] л А к "=" 0

Сравнивая уравнения (016) и (022), заметим, что первое можно было бы вывести из второго, если

(023) л А к "=" А к + ф Икс к , л ( А к ) "=" с 2 ( А Икс к А к ) , л А к "=" Дж к ϵ 0

Из 1-го уравнения (023) л β часть лагранжевой плотности л должны содержать термины

(024) 1 2 А к 2 + ф Икс к А к , к "=" 1 , 2 , 3
и поэтому их сумма по отношению к к
(025) л β 1 1 2 А ˙ 2 + ф А ˙

Из 2-го уравнения (023) л β часть лагранжевой плотности л должны содержать термины

(026) 1 2 с 2 [ А Икс к А к А к 2 ] , к "=" 1 , 2 , 3
и поэтому их сумма по отношению к к
(027) л β 2 1 2 с 2 к "=" 1 к "=" 3 [ А Икс к А к А к 2 ]
Из 3-го уравнения (023) л β часть лагранжевой плотности л должны содержать термины
(028) Дж к А к ϵ 0 , к "=" 1 , 2 , 3
и поэтому их сумма по отношению к к
(029) л β 3 Дж А ϵ 0

Из уравнений (025), (027) и (029) л β часть лагранжевой плотности л является

(030) л β "=" л β 1 + л β 2 + л β 3 "=" 1 2 А ˙ 2 + ф А ˙ + 1 2 с 2 к "=" 1 к "=" 3 [ А Икс к А к А к 2 ] + Дж А ϵ 0

Наконец, из выражений (020) и (030) для плотностей л α , л β плотность Лагранжа л "=" л α + л β является

(031) л "=" л α + л β "=" ( А ) ф + 1 2 ф 2 р ф ϵ 0 + 1 2 А ˙ 2 + ф А ˙ + 1 2 с 2 к "=" 1 к "=" 3 [ А Икс к А к А к 2 ] + Дж А ϵ 0 (это неправильная плотность Лагранжа)

3. Ошибка-проба-окончательный успех

Вставка этого выражения плотности Лагранжа в уравнение Лагранжа относительно ф , то есть уравнение (017), не дает уравнения (006), но

(032) 2 ф т ( 2 А ) "=" р ϵ 0 , ( неправильный )
Появление доп. ( А ) происходит из-за термина ( ф А ˙ ) из л β и поэтому плотность Лагранжа, заданная уравнением (031), не является подходящей.

Чтобы решить эту проблему, мы должны посмотреть на (015), то есть (006), с другой точки зрения следующим образом.

(033) ( ф + А ˙ ) ( р ϵ 0 ) "=" 0

Сравнивая уравнения (033) и (017), заметим, что первое можно было бы вывести из второго, если вместо (018) иметь

(034) л ф "=" 0 , л ( ф ) "=" ф + А ˙ , л ф "=" р ϵ 0

поэтому вместо (019) и (020) соответственно уравнения
(035) л α 1 0 , л α 2 1 2 ф 2 + ф А ˙ , л α 3 "=" л α 3 р ф ϵ 0
(036) л α "=" л α 1 + л α 2 + л α 3 "=" 1 2 ф 2 + ф А ˙ р ф ϵ 0
Теперь необходимо исключить из л β 1 , уравнение (025), второй член ( ф А ˙ ) так как он появляется в л α 2 , см. второе из приведенных выше уравнений (035).

Итак, мы имеем вместо (025)

(037) л β 1 1 2 А ˙ 2
пока л β 2 , л β 3 остаются неизменными, как в уравнениях (027) и (029)
(038) л β 2 "=" л β 2 1 2 с 2 к "=" 1 к "=" 3 [ А Икс к А к А к 2 ] (039) л β 3 "=" л β 3 Дж А ϵ 0

Вместо (030)

(040) л β "=" л β 1 + л β 2 + л β 3 "=" 1 2 А ˙ 2 + 1 2 с 2 к "=" 1 к "=" 3 [ А Икс к А к А к 2 ] + Дж А ϵ 0
и, наконец, для новой лагранжевой плотности мы имеем вместо (031)

(041) л "=" л α + л β "=" 1 2 ф 2 + ф А ˙ р ф ϵ 0 + 1 2 А ˙ 2 + 1 2 с 2 к "=" 1 к "=" 3 [ А Икс к А к А к 2 ] + Дж А ϵ 0

Плотность л (041) получается из плотности л уравнения (031), если опустить член ( А ) ф . Так л не зависит от ф .

В следующих уравнениях фигурная скобка над левыми тремя членами группирует ту часть плотности л который существенно участвует в производстве электромагнитного уравнения (006) из уравнения Лагранжа относительно ф , уравнение (017), а фигурная скобка под правыми 4 членами группирует ту часть плотности л который существенно участвует в производстве электромагнитных уравнений (008) из уравнений Лагранжа относительно А 1 , А 2 , А 3 , уравнение (022).

л "=" 1 2 ф 2 р ф ϵ 0 + ф А ˙ в отношении  ф + 1 2 А ˙ 2 + 1 2 с 2 к "=" 1 к "=" 3 [ А Икс к А к А к 2 ] + Дж А ϵ 0

л "=" 1 2 ф 2 р ф ϵ 0 + ф А ˙ + 1 2 А ˙ 2 + 1 2 с 2 к "=" 1 к "=" 3 [ А Икс к А к А к 2 ] + Дж А ϵ 0 в отношении  А

Обратите внимание на общий термин ( ф А ˙ ) .

Переупорядочив слагаемые в выражении (041) плотности л у нас есть

л "=" 1 2 А ˙ 2 + 1 2 ф 2 + ф А ˙ 1 2 ф А т 2 1 2 с 2 к "=" 1 к "=" 3 [ А к 2 А Икс к А к ] × А 2 + 1 ϵ 0 ( р ф + Дж А )
(042)

то есть

(043) л "=" 1 2 | | ф А т | | 2 1 2 с 2 | | × А | | 2 + 1 ϵ 0 ( р ф + Дж А )
или
(044) л "=" | | Е | | 2 с 2 | | Б | | 2 2 + 1 ϵ 0 ( р ф + Дж А )

Теперь, если плотность л должны иметь размеры энергии на единицу объема, которые мы определяем л е м "=" ϵ 0 л так

(045) л е м "=" ϵ 0 | | Е | | 2 с 2 | | Б | | 2 2 р ф + Дж А
имея в виду, что
(046а) Е 2 "=" ф А т 2 "=" А ˙ 2 + ф 2 + 2 ( ф А ˙ ) (046б) Б 2 "=" × А 2 "=" к "=" 1 к "=" 3 [ А к 2 А Икс к А к ]

скаляр ( | | Е | | 2 с 2 | | Б | | 2 ) является одним из двух инвариантов Лоренца (2) поля (другой Е Б ) практически равно постоянному времени Е мю ν Е мю ν , где Е мю ν антисимметричный тензор поля (2) .

С другой стороны, скаляр ( р ф + Дж А ) по сути является внутренним продуктом Дж мю А мю в пространстве Минковского двух 4-векторов: 4-плотность тока Дж мю "=" ( с р , Дж ) и 4-потенциал А мю "=" ( ф / с , А ) , тоже лоренц-инвариантный скаляр.

Итак, лагранжева плотность л е м в уравнении (045) является лоренц-инвариантным.


(1) Путем проб и ошибок я нашел лагранжиан в более сложном и сложном случае: см. мой ответ как user82794 здесь Получите лагранжиан из системы связанных уравнений

(2) Следуя У. Риндлеру в «Введении в специальную теорию относительности» изд. 1982 г., этот тензор выводится из уравнения (38.15)

(38.15) Е мю ν "=" [ 0 Е 1 Е 2 Е 3 Е 1 0 с Б 3 с Б 2 Е 2 с Б 3 0 с Б 1 Е 3 с Б 2 с Б 1 0 ] так Е мю ν "=" [ 0 Е 1 Е 2 Е 3 Е 1 0 с Б 3 с Б 2 Е 2 с Б 3 0 с Б 1 Е 3 с Б 2 с Б 1 0 ]
что, делая замены (двойственности) Е с Б и с Б Е урожаи
(39.05) Б мю ν "=" [ 0 с Б 1 с Б 2 с Б 3 с Б 1 0 Е 3 Е 2 с Б 2 с Е 3 0 Е 1 с Б 3 Е 2 Е 1 0 ] так Б мю ν "=" [ 0 с Б 1 с Б 2 с Б 3 с Б 1 0 Е 3 Е 2 с Б 2 с Е 3 0 Е 1 с Б 3 Е 2 Е 1 0 ]
Два инварианта Е мю ν - сразу распознаваемые как таковые по способу их образования - могут быть выражены следующим образом:
(39.06) Икс "=" 1 2 Е мю ν Е мю ν "=" 1 2 Б мю ν Б мю ν "=" с 2 | | Б | | 2 | | Е | | 2 (39.07) Д "=" 1 4 Б мю ν Е мю ν "=" с Б Е

В чем разница между "пробами и ошибками" и "угадыванием"?
Красивый латекс
Профессор факультета электротехники и вычислительной техники Университета Нью-Мексико по имени Бахман Зохури опубликовал 30 января 2019 года статью в формате pdf под названием «Вывод лагранжевой плотности электромагнитного поля» с точной копией этого ответа. здесь. Жаль, что в его ссылках нет ни слова о PhysicsStackExchange и моем ответе. Ссылка : Вывод лагранжевой плотности электромагнитного поля
Трудно поверить, что профессор из «известного» университета скопировал ваш мир, даже не упомянув об этом. Что более удивительно, так это то, что ее опубликовало «известное» издательство, по меньшей мере отвратительное.
@renormalizedQuanta : ...действительно, трудно поверить...
Этот ответ великолепен! (+++1)
@Frobenius, по поводу этой копии-вставки, я думаю, было бы лучше, если бы кто-нибудь связался со Springer и сообщил об этом плагиате. Престижные журналы часто обвиняют профессоров в плагиате. Я почти уверен, что он делает это не в первый раз и не в последний раз.
В тот момент, когда я подумал, что вы должны опубликовать это, я увидел, что профессор использовал это!
@Deschele Schilder: ... что я могу сказать об этом. Как видите, я отправляю этот комментарий 4 июня 1919 года и не отвечаю. Я уверен, что профессор из "известного" университета и "известного" издательства знают об этом. Кстати, всегда приятно видеть здесь такие комментарии. Математически нелогичный аргумент в выводе уравнения Гамильтона у Гольдштейна .
Как в уравнении (016) мю 0 Дж идти к Дж к ϵ 0 ?
@moboDawn_φ: Добро пожаловать в PSE. Потому что
с 2 ( 008 ) "=" ( 016 ) с 2 мю 0 Дж "=" Дж ϵ 0
Спасибо за этот удивительный ответ и работу!
@abu_bua: Добро пожаловать. Предлагаю заглянуть и сюда. Почему сложное скалярное поле и его комплексно-сопряженное поле считаются двумя разными полями? для лагранжевой плотности уравнения Шредингера.
@Frobenius вы сообщили редактору/издателю?
@ziyuang: Спасибо за внимание. Нет, я не отчитывался. Не думаю, что стоит это делать.
@Frobenius Хороший ответ! Хотя бы один комментарий! Уравнения Максвелла, которые вы написали, не являются уравнениями Максвелла в пустом пространстве. Они находятся в присутствии зарядов и токов ( р , Дж 0 ).

В конечном счете, рассуждение должно заключаться в том, что (как вы заявили) оно должно быть построено так, чтобы уравнения Эйлера-Лагранжа были уравнениями Максвелла. Таким образом, в некотором смысле вы должны угадать лагранжиан, который производит это, как это сделано, например, здесь .

Однако вы можете получить некоторое представление из того факта, что нам нужно построить лагранжиан для безмассового несамодействующего поля. Поэтому нам нужна калибровочная и лоренц-инвариантная комбинация 4-векторного потенциала, который имеет только кинетический член (квадратичный по производным полей). Тогда у вас не останется много вариантов, кроме Ф мю ν Ф мю ν . Исходный термин затем легко добавить, если это необходимо.

Как насчет ϵ мю ν о т Ф мю ν Ф о т ?
Ну, я сказал «не осталось много вариантов», и действительно, комбинация, которую вы записываете, также соответствовала бы моим критериям, как и дет ( Ф ) , но можно попробовать сначала самый простой вариант, и он окажется правильным. Я отмечаю, что указанная вами комбинация является псевдоскаляром. Может ли кто-нибудь подумать, есть ли причина, по которой это было бы запрещено?
@MistakeInk - ϵ мю ν р о Ф мю ν Ф р о является прекрасным термином-кандидатом для лагражиана, просто это полная производная, поэтому она не влияет на классический EOM и исчезает в теории возмущений. Однако это все же имеет некоторые последствия — см. en.wikipedia.org/wiki/CP_violation#Strong_CP_problem . Что касается Д е т ( Ф ) Я не думаю, что этот термин перенормируем, поскольку он равен е т р бревно Ф которую можно разложить на некоторое значение фонового поля и получить сколь угодно высокие степени напряженности поля.
Однако вы получаете условия формы бревно т р ( к 2 + Ф мю ν 2 ) при расчете эффективного действия при наличии фонового поля калибровочного поля. См. главу 16 Пескина.
@DJBunk Вы правы, что дет ( Ф ν мю ) является неперенормируемым, поэтому он не будет работать, но он только четвертой степени в Ф и не содержит произвольно больших степеней. Он также не является лоренц-инвариантным.
@DJBunk Ненулевое значение ϵ Ф Ф термин реально реализован в реальных экспериментальных системах - трехмерных топологических изоляторах. Это не влияет на физику объема, но порождает топологически защищенные бесщелевые краевые моды на границе системы.

Я почти на 100% уверен, что лагранжиан является предположением теории. Его нельзя вывести. У меня нет ссылок на это утверждение. Я просто знаю, что из каждого курса, который мне преподавали, и из каждой книги, которую я читал, лагранжиан (при условии, что он вообще используется) - это то, с чего вы начинаете. В данном случае это «первый принцип».

Спасибо - я думаю, что я когда-либо видел лагранжианы только из механики, где они, естественно, имеют форму л "=" Т В и, следовательно, то, что я называл «производным».
Я тоже не понимаю, как этот лагранжиан «выводим». Конечно, мы пишем его так, чтобы уравнения Эйлера-Лагранжа давали нам классические уравнения движения. Но я бы не считал это выводом. На самом деле это просто замена одного предположения другим.

Ответ вы можете найти в книге «Дифференциальная геометрия и группы Ли для физиков» Мариана Фецко.

На геометрическом языке действие поля Ф е Ом п ( М ) на некотором n-мерном римановом многообразии ( М , г ) следует понимать как «внутренний продукт»

М Ф * г Ф ,
где п < н , и * г является оператором звезды Ходжа, т.е.
* г : Ом п ( М ) Ом н п ( М )
так что действие инвариантно к диффеоморфизму и его плотность является скаляром.

Например, действие свободного скалярного поля принимает следующий вид:

С [ ф ] "=" М г ф * г г ф + м 2 М ф * г ф "=" М | г | г н Икс { мю ф мю ф + м ф 2 2 } .
Когда многообразие «мировой лист» является 1-мерным, только возможные поля являются 1-формами. Можно считать, что действие принимает следующий вид
р А мю г Икс мю г с г с
где А "=" А с г с "=" А мю г Икс мю г с г с является 1-формой на мировой линии, дуальное поле звезды Ходжа которой не определено.

Вы можете использовать симметрии E&M, чтобы показать, что существует только один разумный кандидат, которого необходимо проверить:

Искомое действие должно быть лоренц-инвариантным, калибровочно-инвариантным, инвариантным по четности и обращению времени и не выше второго порядка по производным. Единственный кандидат — [действие Максвелла]. [Средницкий QFT стр. 334.]